Write an explicit formula for each sequence given below.
Part A
1,-10, 100,-1,000, 10,000
Explicit Formula: In =
Part B
10.2, 2.2, -5.8, -13.8, -21.8
Explicit Formula: an =

Write An Explicit Formula For Each Sequence Given Below.Part A1,-10, 100,-1,000, 10,000Explicit Formula:

Answers

Answer 1

Answer:

The first five values are 12, 14, 16, 18 and 20

The recursive formula is as follow

Step-by-step explanation:

Since we need to find the first 5 values  we start with n = 1 to n = 5. Note that we don't start from n = 0 as the question states that  n ≥ 1.

For the first 5 values we just replace the value of n with 1, 2, 3, 4 and 5 and calculate the answer

The first five values are 12, 14, 16, 18 and 20

A recursive formula is one that

a) Mentions the initial term

b) provides a formula connecting the previous term to the existing term.

Since we know the first term is 12, i.e  and we know that the difference between consecutive terms is 2 we can conclude that the recursive formula is made up of the following two formulas

The over all formula is as follow

Step-by-step explanation:

Hope this helps:)


Related Questions

Answer the question based on this system of equations.

{x=y−5
6x−2y=22

Which of the following choices help to verify whether (7,12) is a solution to the system above?

Select all that apply.

Group of answer choices

1.) 7=12−5 is true.

2.) 7=12−5 is not true.

3.) 6(7)−2(12)=22 is true.

4.) 6(7)−2(12)=22 is not true.

Answers

Step-by-step explanation:

1.) 7=12 - 5 is true

4.) 6(7) - 2(12) = 22 is not true

Help help math please help help help help

Answers

Answer:

Answered is x= 55x

Step-by-step explanation:

13x+10 * 12x+20

soln:

or, 13x+12x+10+20

or, 25x+30

:., therefore, x=55x

plsss answer my questions in the picture

Answers

Answer:

8. B

9. B

10. B

thats my answer

Dan bought a truck for $29,800. The value of the truck depreciated at a constant rate per year. The table shows the value of the truck after the first and second years: Year 1 2 Value (in dollars) 26,522 23,604.58 Which function best represents the value of the truck after t years? f(t) = 29,800(0.89)t f(t) = 26,522(0.89)t f(t) = 29,800(0.11)t f(t) = 26,522(0.11)t

Answers

Answer:

A) f(t) = 29,800(0.89)^t

Step-by-step explanation:The truck value decreases by 11% each year.

The present value of the truck was $29,800.

29,800 x 0.11 = 3,728 dollars lost from value.

29,800 - 3,728 = 26,522, the value of the truck after one year.

26,522 x 0.11 = 2,917.42 cash lost from value.

26,522 - 3,728 = 23,604.58, the value of the truck after two years.

Therefore, the answer is A) f(t) = 29,800(0.89)^t.

Area of a circle
You might need:
Calculator
What is the area of the following circle?
Either enter an exact answer in terms of or use 3.14 for it and enter your answer as a decimal.
T = 1
units

Answers

Answer:

If the radius is 1, then the answer is pi.

Radius=r=1

Area

πr²π(1)²1ππ3.14 units²

Answer: 3x10^2 mi

example:

Answers

Answer:

3000

Step-by-step explanation:

Answer:

300

Step-by-3step explanation:

1.
Write the equation of the parabola in vertex form.



A. y = –(x – 1)2 + 3

B. y = –x2 – 4

C. y = –x2 + 3

D. y = –x2 + 4

Answers

[tex]~~~~~~\textit{vertical parabola vertex form} \\\\ y=a(x- h)^2+ k\qquad \begin{cases} \stackrel{vertex}{(h,k)}\\\\ \stackrel{"a"~is~negative}{o pens~\cap}\qquad \stackrel{"a"~is~positive}{o pens~\cup} \end{cases} \\\\[-0.35em] \rule{34em}{0.25pt}[/tex]

[tex]\begin{cases} h= 0\\ k = 4 \end{cases}\implies y=a(x-0)^2+4~\hfill \textit{we also know that} \begin{cases} x = 1\\ y = 3 \end{cases} \\\\\\ 3=a(1-0)^2+4\implies 3=1a+4\implies \boxed{-1=a} \\\\\\ y=-1(x-0)^2+4\implies \blacktriangleright y = -x^2+4\blacktriangleleft[/tex]

Answer:

A.    y = –(x – 1)2 + 3    = Rewrite in vertex form and use this form to find the vertex  (h,k).    (1,3)=  Already in vertex form.  y=−(x−1)2+3

OR

D.    y = –x^2 + 4    = Rewrite in vertex form and use this form to find the vertex  (h,k).    (0,4)=  Find the vertex form.     y=−(x+0)2+4

Step-by-step explanation:

A.    y = –(x – 1)2 + 3    = Rewrite in vertex form and use this form to find the vertex  (h,k).    (1,3)=  Already in vertex form.  y=−(x−1)2+3

B.    y = –x^2 – 4    = Rewrite in vertex form and use this form to find the vertex  (h,k).    (0,−4)=   Find the vertex form.     y=−(x+0)2−4

C.    y = –x^2 + 3     = Rewrite in vertex form and use this form to find the vertex  (h,k).    (0,3)=  Find the vertex form.     y=−(x+0)2+3

D.    y = –x^2 + 4    = Rewrite in vertex form and use this form to find the vertex  (h,k).    (0,4)=  Find the vertex form.     y=−(x+0)2+4

Which inequality represents this sentence?
A number is no less than 48.
n < 48
n > 48
n < 48
n>48

Answers

The inequality that can be used to represent the information that a number is no less than 48 will be n > 48.

Inequality simply means a mathematical expression that is used in which the sides aren't equal to each other.

It should be noted that the expression that a number is no less than 48 means the number is greater than 48. Therefore, the inequality is n > 48.

Read related link on:

https://brainly.com/question/25701252

Answer: The correct answer is n≥48

Step-by-step explanation: Give me the brainiest

Classify: −4m+21(m−1)≤15m+2(m−1).

Answers

Answer:

[tex]\boxed{\sf All\: real\: numbers}[/tex]

Step-by-step explanation:

[tex]\star \:\sf Classify\: -4m+21\left(m-1\right)\le \:15m+2\left(m-1\right)\: \star[/tex]

Apply Distribution law:

[tex]\sf -4m+21m-21\le \:15m+2\left(m-1\right)[/tex]

Combine like terms:

** [tex]\sf \hookrightarrow -4m+21m=17m[/tex]

[tex]\longmapsto\sf 17m-21\le \:15m+2\left(m-1\right)[/tex]

[tex]\sf 2(m-1)[/tex]

[tex]\hookrightarrow 2m-2[/tex]

-----------------------------------------

[tex]\longmapsto\sf 17m-21\le \:15m+2m-2[/tex]

Combine like terms:

** [tex]\sf 15m+2m=17m[/tex]

[tex]\longmapsto\sf 17m-21\le \:17m-2[/tex]

Add 21 from both sides:

[tex]\longmapsto\sf 17m-21+21\le \:17m-2+21[/tex]

[tex]\longmapsto\sf 17m\le \:17m+19[/tex]

Subtract 17m from both sides:

[tex]\longmapsto\sf 17m-17m\le \:17m+19-17m[/tex]

[tex]\longmapsto\sf 0\le \:19[/tex]

Therefore, the solution of the inequality: All real numbers

_____________________________________________

Solve. 140% of 340 is what number?​

Answers

Answer:

The answer is 476

Step-by-step explanation:

hope this helps :)


I NEED HELP W THIS ASAP!!!!!

Answers

Answer:

[tex]18[/tex]

Step-by-step explanation:

[tex] \frac{28}{14} = 2 \\ \frac{36}{x} = 2 \\ x = \frac{36}{2} \\ = 18[/tex]

The problem 7,291 ÷ 19 is solved below using partial quotients. Identify what partial quotient was used in each step. Then, identify your final quotient.

Seven thousand two hundred ninety one minus five thousand seven hundred equals one thousand five hundred ninety one minus one thousand five hundred twenty equals seventy one minus fifty seven equals fourteen. Arrows pointing to step one, two, and three. You can not make anymore groups of nineteen. What is left becomes your remainder ill give brainliest

Answers

Using the partial quotient method, it is found that:

The partial quotients used in each step are as follows: 300, 80, The final quotient is of: 383.

Partial quotient method

When division is done applying the partial quotient method, consecutive subtractions are done with n groups of the divisor.

Each partial quotient is the number of groups, and the final quotient is the sum of the partial quotients.

In this problem, the divisor is as follows;

19.

The first subtraction is by 5700, which is 5700/19 = 300 groups of 19, hence the first partial quotient is of 300.

The second subtraction is by 1520, which is 1520/19 = 80 groups of 19, hence the second partial quotient is of 80.

The third subtraction is by 57, which is 57/19 = 3 groups of 19, hence the third partial quotient is of 3.

Then the final quotient is given as follows:

300 + 80 + 3 = 383.

Missing Information

The subtractions are given by the image at the end of the answer.

More can be learned about the partial quotient method at https://brainly.com/question/2059458

#SPJ1

6% of a value is 510

Answers

Answer:

The value is 8500

Step-by-step explanation:

Of means multiply and is means equals

6% * V = 510

Change percent to decimal form

.06V = 510

Divide each side by .06

V = 510/.06

V =8500

Answer:

9

(6 ÷ 100) 150

6÷2×3

9

ans 9

Two hundred fifty tickets are sold at a fund raiser. If Sarah buys 15 tickets, the probability that she will win the prize quilt is
.

Answers

Answer:

6%

Step-by-step explanation:

15/250=3/50   or 6%

The probability that Sarah would win the prize quilt given the tickets she bought  is 6%.

What is the probability?

Probability is how likely it is that a stated event would happen. The probability of the event happening lie between 0 and 1.

The probability that Sarah would win the prize quilt = number of tickets bought / total number of tickets

(15 / 250) x 100 = 6%

To learn more about probability, please check: https://brainly.com/question/13234031

#SPJ2

3/8 + 1/8 + 2/3 - 1/2 =

Answers

Answer: 2/3 (answer in decimal form = 0.6)

Step-by-step explanation: The least common multiple (LCM) of two or more non-zero whole numbers is the smallest whole number that is divisible by each of those numbers. In other words, the LCM is the smallest number that all of the numbers divide into evenly. Then do the rest of the operations.

QUICK! I NEED HELP ASAP
---------------------
w−(−1/3)=2.05

Enter your answer as a mixed number in simplest form in the box.

w =

Answers

Answer:

w=6

Step-by-step explanation:  you should just look it up on line

Answer:

The correct answer is 1 43/60

Step-by-step explanation:

you convert 2.05 to a fraction and get 2 23/60 - (-1/3)  = 1 43/60

i also took the test

Simplify the expression 2.1(3.7 +5.7u) using the Distributive Property.
PLEASE HELP! :(​

Answers

Answer:

7.77+11.97u

Step-by-step explanation:

2.1(3.7+5.7u)=7.77+11.97u

What is the equation of this line in slope-intercept form? (-1,5) (1,-1)




y = 3x - 2

y = -1/3x + 2

y = -3x + 2

y = 3x + 2

Answers

Answer:

C

Step-by-step explanation:

y=-3x+2

Collect the like terms: 5a + 2b + b - 2a​

Answers

Answer:

5a+2b+b-2a

5a-2a+2b+b

3a+3b

Adrienne biked 24 miles in 5 1/2 hours. If she biked at a constant speed, how many miles did she ride in one hour?

1. How can you solve this?

Answers

Answer:

4.36

Step-by-step explanation:

dividing the amount of hours by how many miles Adrienne went will give you how many miles she went per hour.

Equation:

a÷b

a=24

b=5.5 (5 1/2)

24÷5.5=4.36

Select the graph which correctly displays the function f(x) = |x + 2|– 3.

graph 1
graph 2
graph 3
graph 4

Answers

Answer:

Graph 4

Step-by-step explanation:

I have to put more words lol

pls help me i will give u thanks

Answers

(+6,+5)
Hope this helps!!

Answer:

c) (6,5)

Step-by-step explanation:

The horizontal axis represents x values

The vertical axis represents y values

Two angles are supplementary. One of the angles measures 4(2x+9)and the other angle measures 2(4x−8). What is the value of x?

Answers

Answer:

x=10

Step-by-step explanation:

4(2x+9) +2(4x-8) = 180 >> sumpplementary angles .

16x = 180 -36+16

16x=160

x=10

Select the correct answer from the drop-down menu.
Rewrite the formula to find the radius of a sphere.
V=
= GT 73
The volume (V) of a sphere is given by the formula
The sphere's radius (() is
Reset
(3V/4pi)1/3
(4V/3pi)1/3
(3pi/4V)1/3

Answers

Answer:

The formula of a sphere is v= 4/3 pi root to the power of 3.

Step-by-step explanation:

The answer is B

Pre college need help

Answers

Answer:

C

Step-by-step explanation:

Given slope of −25 with point (-5,3), write the linear equation in Slope-Intercept Form.

Answers

[tex]\\ \sf\longmapsto y=mx+b[/tex]

[tex]\\ \sf\longmapsto 3=-25(-5)+b[/tex]

[tex]\\ \sf\longmapsto 3=125+b[/tex]

[tex]\\ \sf\longmapsto b=-122[/tex]

Equation of the line

[tex]\\ \sf\longmapsto y=-25x-122[/tex]

$4800 is 60% of what number?

Answers

Answer:

8000

Step-by-step explanation:

Please help me, thanks

Answers

Answer:

D how many months until the total spent is the same for both gyms

Step-by-step explanation:

30m+50 = 10m+100

By setting the cost of gym A equal to gym B we are finding  how many months it is when the cost is the same

Answer:

How many months until the total spent is same for both the gyms

Step-by-step explanation:

Then

GYmA=GYm B30m+50=10m+100

We may solve it

20m=50m=5/2m=2.5months

option D

Sierra is getting a loan to purchase a car, but she has a few options for loan terms. She is comparing the cost of each loan to determine which loan will cost the least in terms of interest paid. (Interest is the fee paid for the use of borrowed money, and it is expressed as a percentage of the amount borrowed multiplied by the amount of time the borrower takes to repay the loan.) All of the loan offers are for $10,000, and include an upfront fee of $200. They are all simple interest loans (as opposed to compound interest loans). Loan A is to be paid back over five years with an annual interest rate of 6 percent. Loan B is to be paid back over four years with an annual interest rate of 4 percent. Loan C is to be paid back over three years with an interest rate of 5 percent. How does Sierra figure out which loan will require the lowest interest payments?

Answers

Answer:

B

Step-by-step explanation:

1/4 of students in a class play a musical instrument of those students 2/5 also speak a second language. What fraction of the whole class plays a musical instrument and speak a second language

Answers

Answer:

Step-by-step explanation:

Say the class has 20 students

20(1/4) = 5 play instruments

5(2/5) = 2 speak a second language

2 / 20 = 1/10

Other Questions
Quarrel not nor contend with thy brethren or sisters. mean's? Based on the diagram, can point d be the centroid of triangle acf? explain. Yes, point d is the point of intersection of segments drawn from all three vertices. Yes, de is three-quarters of the length of the full segment. No, de should be longer than ad. No, the ratio between ad and de is 3:1. Paying 13 mortgage payments instead of 12 per year can save to you thousands in interest cost. If you had a $175,000 mortgage at 6% for 30 years, how much extra would you have to pay per year to make 13 instead of a 12 mortgage mortgage payments per year? How much would you pay if you paid 1/12 of it per month? in the 1950s and 60s, which two television shows helped introduce america to new rock n roll artists by frequently having those artists present live performances on the show? name the opening though which molten rock flowers to the surface Blood vessel diameter fluctuates between being constricted and being dilated. This is the primary contributor to blood pressure. In order for blood vessels (arterioles) to maintain a steady pressure, there must be a prolonged presence of what ion in the smooth muscle cell's cytosol The _____ of a firm uses information from the sales budget and various cost budgets to develop a forecast of net earnings for the planning period. In the general population, 1 in 300 individuals is a carrier for Tay-Sachs disease, while 1 in 30 individuals of Ashkenazi Jew descent are carriers. Tay-Sachs also affects 1 in 30 individuals with French-Canadian ancestry, although two completely unique mutations are responsible for the Ashkenazi and French-Canadian mutations.A) What are the chances of two individuals having a child with Tay-Sachs if both individuals are not of Ashkenazi or Franch-Canadian descent?B) What are the chances of two Individuals having a hild with Tay-Sachs lf one Individual Is of Ashkenazi descent and one ls not? ANSWER ASAPPPPPPPPPPPPPPPPPPPPPPPPPPPPPPPPPP Water leaks from tanker at a rate of 8L/h. How much water will leak from the tank in: a: 2 Day. b: 2 Weeks a group consisting of only the wood, american, cave, and desert cockroaches is a monophyletic group. true or false? Use the words to make sentences.0. He / collect / comics (+)He collects comics.1. What time / they / usually / go to bed (?)2. She / enjoy / reading the newspaper (-) You / brush your teeth / every day (?)He / go / English class / three times / week (+)5. How often / you / take photos (?)6. She / write / a blog about cats (+)7. My cousin / do / any sports (-) A business is having trouble keeping up with the competition. They cannotrespond to their customers like the competition. Which type of system willlikely be able to help them solve this issue?A. Transaction processing systemB. Management communication systemC. Decision support systemD. Management information system Can someone help me please? WILL GIVE BRAINLIST NO LINKS plzzz or FILLESWhat is the climate today in the region of the ancient civilization? i choose Indus river And yellow river valley 100% right will earn brainliest. QuestionSelect the factored form of the expressions 3x211x4A (3x4)(x+1)B (3x2)(x+2)C (3x+1)(x4)D (3x1)(x+4) Find the percent of increase or decrease. Round to the nearest tenth of a percent where necessary.32 to 40. simple math question!!! There are 12 boxes of books to put on shelves. Each box has 31 books.Each shelve holds 18 books. Kevin estimates he will need 20 shelves for all the books